LSAT and Law School Admissions Forum

Get expert LSAT preparation and law school admissions advice from PowerScore Test Preparation.

 Administrator
PowerScore Staff
  • PowerScore Staff
  • Posts: 8916
  • Joined: Feb 02, 2011
|
#66052
Please post your questions below!
 lsatcheetah9876
  • Posts: 3
  • Joined: Jul 09, 2019
|
#66324
Hi! This question stumped me. I picked D, because I don't think I understood the question type.

Honestly I still don't understand what it's asking.
 James Finch
PowerScore Staff
  • PowerScore Staff
  • Posts: 943
  • Joined: Sep 06, 2017
|
#66336
Hi LSAT Cheetah,

This is a type of Strengthen question that is asking for a broad principle rather than a more specific answer. So let's look at what the stimulus is saying and where the logical gap is that we will need to shore up.

The stimulus begins by stating a claim by a campaign, then immediately stating that this claim isn't true. This is a common LSAT logical reasoning stimulus type, the counterargument. This denial that the campaign believes its own statement is the conclusion, and what follows is a premise for why the campaign doesn't believe its own statement. This is the relatively weak evidence that they have had the opportunity to release the quote in context, but have failed to do so. So how to strengthen this? The only evidence we have to go on is that the full context is apparently more damaging, and they haven't released the full context despite having had multiple opportunities, so to Prephrase we could come up with a conditional statement like "if the full context is more damaging, an opposing campaign will always release it at the first opportunity." This gives us a contrapositive of No Release :arrow: Not More Damaging, which would essentially justify the stimulus's conclusion.

The correct answer choice should give us something along these lines, albeit in broader Principle form, so that not releasing the full context makes it likelier that the campaign doesn't believe it's more damaging. So the most accurate Prephrase would be something along the lines of "If more damaging, then will be released/used."

(A) is effectively a restatement of our Prephrase, in more abstract, Principle form: Most Damaging :arrow: Pursue. And because the full context wasn't pursued, we can use the contrapositive to show:

Pursue :arrow: Most Damaging

which basically justifies the conclusion.

Hope this clears things up!
User avatar
 JocelynL
  • Posts: 51
  • Joined: Dec 22, 2020
|
#85022
This question stem completely confused me. The explanation uses conditional reasoning but the question is identified as causal. Can someone please explain this one in the causal form? I thought this was some form of straw man flaw but that really didn't help me identify the correct answer.
 Jeremy Press
PowerScore Staff
  • PowerScore Staff
  • Posts: 1000
  • Joined: Jun 12, 2017
|
#85109
Hi Jocelyn,

The reason James's explanation employs conditional reasoning in the prephrase process is because coming up with a general conditional connection between the premises and conclusion of an argument is a great way to strengthen that argument. That conditional connection might, in some cases, incorporate a causal idea, as it does here.

Consider a hypothetical argument. "Because a predictable study schedule leads to testing success, therefore I should adopt such a schedule in my LSAT studying." A general rule that strengthens that argument might create a conditional connection between the premise and the conclusion by saying, "If a study technique leads to testing success, then it should be adopted." Notice how both the premise of the argument and the sufficient condition of the principle incorporate that causal idea of a study strategy causing (leading to) test success?

The same thing is happening here. The conditional principle that James prephrased is great, and fits answer choice A very well. This is something we see extremely often on Strengthen-Principle questions. The answer states a general conditional rule that can be used to connect the stimulus's premise to its conclusion. The form of that conditional is ordinarily [Premise} :arrow: [Conclusion].

Within the conditional principle here, though, there is a cause and effect relationship, which is also present in the argument in the stimulus. The argument in the stimulus assumes that what is causing the Johnson campaign to quote the opponent out of context is that the Johnson campaign thinks the out-of-context quotation is the most politically damaging form of the quote. In other words, assuming the campaign would be motivated (i.e. caused) to do the most damaging thing possible, the fact that the campaign doesn't ever quote the opponent in context means they think it wouldn't be as politically damaging to quote her in context. Answer choice A incorporates that cause/effect assumption. What will cause a campaign to act in a certain way, according to answer choice A? What will cause them to act that way is whatever they believe to be the most damaging to their opponent politically.

I hope this helps clarify!
 g_lawyered
  • Posts: 211
  • Joined: Sep 14, 2020
|
#95479
Hi PS,
This question gave me a hard time as I realized I completely missed the mark on the argument. I saw 2 speakers in this question. The analyst (who the question directly tells us to support) and Johnson's campaign manager. To clarify this explanation:
This denial that the campaign believes its own statement is the conclusion, and what follows is a premise for why the campaign doesn't believe its own statement. This is the relatively weak evidence that they have had the opportunity to release the quote in context, but have failed to do so.
The counterargument is that Analysts says that Johnson's campaign believes that when Johnson attacked his opponent in the quote, the quote was more damaging to the opponent (than it was to Johnson), correct?
When I solved this question, I understood the counterargument to mean that: The quote WASN'T more damaging to the opponent & instead it was more damaging to Johnson. With my reasoning, I prephrased that to strengthen this I needed a principle that supported that the context of a quote shouldn't be more damaging/protect Johnson (not the opponent). With my line of reasoning, I chose (E). I thought we needed to strengthen what the analyst stated not what the campaign managers are stating. I think this is where I had trouble. Is (E) incorrect because I understood the argument backwards? Is this why (A) is correct instead of (E)? Can someone please break this further down for me? Help :-?
User avatar
 katehos
PowerScore Staff
  • PowerScore Staff
  • Posts: 184
  • Joined: Mar 31, 2022
|
#95494
Hi g_lawyered!

The analyst says that those who run the campaign do not believe the quote was more politically damaging to her in context than out of context because they continue to use the quote out of context. It's not that the quote in context is more damaging to Johnson, but rather to his opponent, which might be why your prephrase was off. We're still missing the link, however, between the analyst's conclusion (that the campaign does not believe the quote is more damaging in context) and the evidence (that the campaign still uses the quote out of context); this missing link is what the question seeks to find. Good job noting that this is what the answer choice should support!

So, we're looking for principles that explain why using the less damaging version of the quote indicates that the campaign does not truly believe it is less damaging to the opponent. Answer choice (E) is incorrect, largely because the principle in (E) doesn't do much to support the analyst's reasoning. We don't know if the quote in context would be damaging to Johnson or open him up to any counterattacks, we only know that the quote in context is - according to Johnson's campaign - more damaging to his opponent. So, avoiding techniques that leave candidates open to counterattacks doesn't support the analyst's conclusion that the campaign does not believe their own statement.

Turning to (A), the principle is that political campaigns will pursue the most damaging attack. If the campaign truly believes that the quote in context is more damaging, then why didn't they use it in context? It doesn't make sense! This helps support the analyst's argument because it shows that the people who run Johnson's campaign do not actually believe it's more damaging

I hope this helps! :)
Kate
User avatar
 mkarimi73
  • Posts: 73
  • Joined: Aug 18, 2022
|
#97479
Can I have a further explanation as to why (D) should be confidently removed from contention? I originally chose (A) because it matched my pre-phrase, and then changed I changed my answer. I got tempted by (D) because I thought it provided a reason as to why Johnson continued to quote out-of-context.

Also, is (A) a conditional statement? Can it be mapped as one? Thanks in advance.
 Adam Tyson
PowerScore Staff
  • PowerScore Staff
  • Posts: 5153
  • Joined: Apr 14, 2011
|
#97879
D should be eliminated because it has no bearing on whether or not those who run the campaign believe what they claimed. That's the conclusion we need to support, so the answer has to address that in some way, like "if they believed that, then they would have done something different."

Answer A is not written as a conditional statement, but it could be expressed in conditional form similar to what I just wrote.

Get the most out of your LSAT Prep Plus subscription.

Analyze and track your performance with our Testing and Analytics Package.